Does Zp Contain Primitive Fourth Roots of Unity?: Investigating p

Click For Summary
The discussion focuses on the conditions under which the group Zp contains primitive fourth roots of unity. It is established that if Zp contains primitive fourth roots of unity, then 4 must divide (p-1). The question arises whether the converse is true: if p ≡ 1 (mod 4), does Zp also contain primitive fourth roots of unity? Participants note that similar reasoning applies to cube roots of unity under the condition p ≡ 1 (mod 3). The conversation emphasizes the relationship between the modulus conditions and the existence of these roots in the group Zp.
Funky1981
Messages
21
Reaction score
0

Homework Statement


p prime, If p=1 ( mod 3) then Zp contains primitive cube roots of unity. Now I am considering which p does Zp contains primitive fourth roots of unity.

opposite way? I mean if p=1(mod4) then Zp contains primitive fourth roots of unity??

2. The attempt at a solution
I can prove that if Zp contains primitive fourth roots of unity, then 4|(p-1) . but how about the opposite way? I mean if p=1(mod4) then Zp contains primitive fourth roots of unity?? I know this statements true if q prime instead of 4. And what values of p does Zp contains primitive fourth roots of unity?
 
Physics news on Phys.org
Funky1981 said:

Homework Statement


p prime, If p=1 ( mod 3) then Zp contains primitive cube roots of unity. Now I am considering which p does Zp contains primitive fourth roots of unity.

opposite way? I mean if p=1(mod4) then Zp contains primitive fourth roots of unity??

2. The attempt at a solution
I can prove that if Zp contains primitive fourth roots of unity, then 4|(p-1) . but how about the opposite way? I mean if p=1(mod4) then Zp contains primitive fourth roots of unity?? I know this statements true if q prime instead of 4. And what values of p does Zp contains primitive fourth roots of unity?

Hi Funky1981! :smile:

The expression ##p \equiv 1 \pmod 3## means that there is a k such that ##p=3k+1##.

Now suppose g is a primitive root mod p.
Then ##g^{\phi(p)} \equiv g^{3k} \equiv 1 \pmod p##.
Therefore ##g^k## is a cube root of 1 in ##\mathbb Z_p##.Same argument holds for ##p \equiv 1 \pmod 4##...
 
Question: A clock's minute hand has length 4 and its hour hand has length 3. What is the distance between the tips at the moment when it is increasing most rapidly?(Putnam Exam Question) Answer: Making assumption that both the hands moves at constant angular velocities, the answer is ## \sqrt{7} .## But don't you think this assumption is somewhat doubtful and wrong?

Similar threads

  • · Replies 1 ·
Replies
1
Views
2K
  • · Replies 4 ·
Replies
4
Views
4K
  • · Replies 3 ·
Replies
3
Views
2K
  • · Replies 14 ·
Replies
14
Views
2K
  • · Replies 1 ·
Replies
1
Views
2K
  • · Replies 5 ·
Replies
5
Views
2K
  • · Replies 17 ·
Replies
17
Views
4K
  • · Replies 1 ·
Replies
1
Views
1K
Replies
8
Views
3K
  • · Replies 6 ·
Replies
6
Views
2K